1. Trang chủ
  2. » Giáo án - Bài giảng

SH hien TM dinh ly cauchy davenport

20 477 3

Đang tải... (xem toàn văn)

Tài liệu hạn chế xem trước, để xem đầy đủ mời bạn chọn Tải xuống

THÔNG TIN TÀI LIỆU

Cấu trúc

  • MỘT SỐ ĐÁNH GIÁ TẬP TỔNG TRÊN Z

  • ĐỊNH LÝ CAUCHY - DAVENPORT

    • Chứng minh thông qua phép biến đổi Davenport

    • Chứng minh bằng đa thức nhiều biến

  • MỘT SỐ ỨNG DỤNG

  • BÀI TẬP TỰ LUYỆN

Nội dung

ĐỊNH CAUCHY - DAVENPORT VÀ ỨNG DỤNG MỘT SỐ ĐÁNH GIÁ TẬP TỔNG TRÊN Z ĐỊNH CAUCHY - DAVENPORT VÀ ỨNG DỤNG Nội dung chuyên đề trình bày số cách chứng minh định Cauchy - Davenport nêu số ứng dụng Để có so sánh khác kết Z Z p (tập số dư theo modulo p, p nguyên tố), phần trình bày số kết quen thuộc tập tổng Z Bạn đọc quan tâm đến kết sâu nội dung tham khảo tài liệu [3] Về kết hay Z p bạn đọc tham khảo kết [1], [2] Trong toàn chuyên đề này, thống ký hiệu A + A = {a + b|a ∈ A, b ∈ B}, c.A = {c.a|a ∈ A} Lời giải số tập tương đối dài, chứa nhiều giải thích quan trọng để bạn đọc hiểu rõ ý tưởng MỘT SỐ ĐÁNH GIÁ TẬP TỔNG TRÊN Z Định 1.1 Cho A tập gồm k số nguyên Khi |2A| ≥ 2k − Nếu A tập gồm k số nguyên |2A| = 2k − A phải cấp số cộng Chứng minh • Đặt A = {a0 , a1 , , ak−1 } với a0 < a1 < a2 < < ak−1 Khi tập 2A chứa k số nguyên 2ai , với i = 0, 1, 2, , k − k − số nguyên ai−1 + , với i = 1, 2, , k − Do 2ai−1 < ai−1 + < 2ai , ∀i = 1, 2, , 2k − Từ suy |2A| ≥ 2k − • Nếu |2A| = 2k − 1, phần tử 2A có dạng 2ai ai−1 + Do ai−1 + < ai−1 + ai+1 < + ai+1 , ai−1 + < 2ai < + ai+1 , ∀i = 1, 2, , k − nên suy 2ai = ai−1 + ai+1 ⇒ − ai−1 = ai+1 − , ∀i = 1, 2, , k − Điều chứng tỏ A cấp số cộng Định chứng minh hồn tồn Tổng qt hơn, ta có đánh giá sau Định 1.2 Cho n ≥ A1 , A2 , , An tập hợp hữu hạn số nguyên Khi |A1 | + |A2 | + · · · + |An | − (n − 1) ≤ |A1 + A2 + · · · + An | ≤ |A1 |.|A2 | |An | Chứng minh • Bất đẳng thức |A1 + A2 + · · · + An | ≤ |A1 |.|A2| |An | hiển nhiên, số biểu diễn dạng a1 + a2 + · · · + an (ai ∈ Ai ) |A1 |.|A2| |An | Ths Trần Minh Hiền(0989.541.123) - Trường THPT chuyên Quang Trung ĐỊNH CAUCHY - DAVENPORT VÀ ỨNG DỤNG MỘT SỐ ĐÁNH GIÁ TẬP TỔNG TRÊN Z • Ta chứng minh bất đẳng thức lại quy nạp Với n = 2, đặt A1 = {a0 , a1 , , ak−1 }, A2 = {b0 , , bl−1 } với a0 < a1 < < ak−1 , b0 < b1 < < bl−1 Giả sử |A1 | = k ≤ l = |A2 | Khi tập A1 + A2 chứa 2k − + (l − k) phần tử phân biệt sau a0 + b0 < a0 + b1 < a1 + b1 < a1 + b2 < · · · < + bi < + bi+1 < ai+1 + bi+1 < · · · < ak−1 + bk−1 < ak−1 + bk < ak−1 + bk+1 < · · · < ak−1 + bl−1 Do |A1 + A2 | ≥ (2k − 1) + (l − k) = |A1 | + |A2 | − Vậy n = định Giả sử kết luận cho ≤ n − tập hữu hạn số số Khi với n tập |A1 | + |A2 | + · · · + |An | = |(A1 + A2 + · · · + An−1 ) + An| ≥ |A1 + A2 + · · · + An−1 | + |An| − ≥ |A1 | + |A2| + · · · + |An−1| − (n − 2) + |An| − = |A1 | + |A2| + · · · + |An| − (n − 1) Chứng tỏ kết luận với n Theo phương pháp quy nạp tốn cho với n tập hữu hạn số nguyên Bổ đề 1.3 Cho A, B hai tập hợp hữu hạn số nguyên |A| = |B| = k Nếu |A + B| = |A| + |B| − A B hai cấp số cộng có cơng sai Chứng minh Đặt A = {a0 , a1 , a2 , , ak−1 } B = {b0 , b1 , b2 , , bk−1 } với a0 < a1 < < ak−1 , b0 < b1 < < bk−1 Khi A + B chứa dãy gồm 2k − số nguyên tăng dần sau a0 + b0 < a0 + b1 < a1 + b1 < a1 + b2 < · · · < ai−1 + bi < + bi < + bi+1 + ai+1 + bi+1 < · · · < · · · < ak−1 + bk−1 Do |A| + |B| = 2k − 1, suy dãy số nguyên chứa toàn phần tử tập A + B Do ai−1 + bi < + bi < + bi+1 ai−1 + bi < ai−1 + bi+1 < + bi+1 dẫn đến ai−1 + bi+1 = + bi ⇒ − ai−1 = bi+1 − bi , ∀i = 1, 2, , k − (1) Tương tự, bất đẳng thức ai−1 + bi−1 < ai−1 + bi < + bi Ths Trần Minh Hiền(0989.541.123) - Trường THPT chuyên Quang Trung ĐỊNH CAUCHY - DAVENPORT VÀ ỨNG DỤNG ĐỊNH CAUCHY - DAVENPORT ai−1 + bi−1 < + bi−1 < + bi dẫn đến ai−1 + bi = + bi−1 ⇒ − ai−1 = bi − bi−1 , ∀i = 1, 2, , k − (2) Từ (1) (2), đặt q = a1 − a0 − ai−1 = bi − bi−1 = q, ∀i = 1, 2, , k − Chứng tỏ A, B hai cấp số cộng có cơng sai q Tổng qt ta có kết sau Bạn đọc quan tâm chứng minh tham khảo bổ đề 1.3, trang 10, tài liệu tham khảo [3] Định 1.4 Cho A, B hai tập hữu hạn số nguyên, với |A| = k ≥ 2, |B| = l ≥ Nếu |A + B| = k + l − A, B hai cấp số cộng có cơng sai ĐỊNH CAUCHY - DAVENPORT Trong phần này, khơng thích thêm tất tập hợp lấy tập trường Z p , tập chứa hệ thặng dư đầy đủ modulo p Định 2.1 (Định Cauchy - Davenport) Cho p số nguyên tố A, B hai tập Z p Khi |A + B| ≥ min{p, |A| + |B| − 1} 2.1 Chứng minh thông qua phép biến đổi Davenport Cho B tập Z p , ∈ B, |B| ≥ A + B = Z p Khi ta có số nhận xét sau: • A + B tập thực A + 2B Thật vậy, với x ∈ A + B, x = a + b(a ∈ A, b ∈ B) ⇒ x = a + b + ∈ A + 2B ∈A ∈B ∈B Vậy A + B ⊆ A + 2B Mặt khác A + B = A + 2B ta có A + 2B = A + 3B Thật vậy: – x ∈ A+2B x = a+b1 +b2 (a ∈ A, b1 , b2 ∈ B) nên x = a + b1 + b2 + ∈ A+3B ∈A Do x ∈ A + 3B nên A + 2B ⊂ A + 3B ∈B ∈B ∈B – x ∈ A + 3B x = a + b1 + b2 + b3 (a ∈ A, b1 , b2 , b3 ∈ B) Do a + b1 + b2 ∈ A + 2B mà A + 2B = A + B nên xảy a + b1 + b2 = a′ + b′(a′ ∈ A, b′ ∈ B) Khi x = a + b1 + b2 + b3 = a′ + b′ + b3 ∈ A + 2B Do A + 3B ⊂ A + 2B ∈A ∈B ∈B Ths Trần Minh Hiền(0989.541.123) - Trường THPT chuyên Quang Trung ĐỊNH CAUCHY - DAVENPORT VÀ ỨNG DỤNG ĐỊNH CAUCHY - DAVENPORT Và hoàn toàn tương tự ta có A + B = A + 2B = A + 3B = · · · Nhưng với a ∈ A = b ∈ B (b tồn |B| ≥ 2) a + nb ∈ A + B, ∀n = 0, 1, 2, Điều phải xảy A + B = Z p , trái với giả thiết • Đặt X = (A + 2B)\(A + B), X = / Đặt Bx = B\B∗x B∗x = {b ∈ B|x − b ∈ A + B}, B∗x X A+B x Bx A + 2B B Khi tập Bx gọi biến đổi Davenport tập B – Khi ∈ B∗x = / Thật vậy, ∈ B∗x x − ∈ A + B hay x ∈ A + B, mâu thuẫn x ∈ X Ngoài B∗x = 0/ với x ∈ X, x = a + b1 + b2 , ∈A ∈B ∈B x − b1 = a + b2 ∈ A + B ⇒ b1 ∈ B∗x – Ta có ∈ Bx ⊂ B, (A + Bx) ∪ (x − B∗x ) ⊂ A + B (1) (tính chất hiển nhiên định nghĩa hai tập Bx , B∗x ) Ngồi ta có (A + Bx ) ∩ (x − B∗x ) = 0/ (2) Thật vậy, giả sử có a + bx = x − b∗x ⇒ x − bx = a + b∗x ∈ A + B ∈Bx ∈B∗x ∈A ∈B∗x ⊂B Do x − bx ∈ A + B ⇒ bx ∈ B∗x (theo định nghĩa B∗x ), mâu thuẫn với bx ∈ Bx – Từ (1) (2) ta có |A + B| ≥ |A + Bx | + |x − B∗x | Vì |x − B∗x | = |B∗x | = |B| − |Bx | nên ta |A + B| ≥ |A + Bx | + |B| − |Bx | (∗) Chứng minh định 2.1 Giả sử định khơng đúng, tồn hai tập A, B Z p mà A + B = Z p (3) |A + B| ≤ |A| + |B| − (4) Kết (3), (4) không thay đổi thay B −b + B (với b ∈ B tùy ý) (Thật vậy, (3) suy phần tử A + B không lập thành hệ thặng dư modulo p, phần tử tập A + (−b + B) chẳng qua phần tử tập A + B tịnh tiến sang trái −b đơn vị nên lập thành hệ thặng dư đầy đủ modulo p Còn tính chất (4) hiển nhiên thay B −b + B | − b + B| = |B|) Do đó, ta giả sử ∈ B Trong tất cặp A, B thỏa mãn (3), (4) ta chọn cặp A, B mà |B| nhỏ Khi |B| ≥ (vì |B| ≤ 1, |A + B| = |A|, |A| + |B| − ≤ |A| − (4) khơng Ths Trần Minh Hiền(0989.541.123) - Trường THPT chuyên Quang Trung ĐỊNH CAUCHY - DAVENPORT VÀ ỨNG DỤNG ĐỊNH CAUCHY - DAVENPORT xảy ra.) Vậy ta có ∈ B, |B| ≥ 2, A + B = Z p Do tồn phép biến đổi Davenport Bx B để (*) xảy Do ≤ |Bx | < |B| Do Bx ⊂ B nên A + Bx ⊂ A + B, suy |A + Bx | < |A + B| < p Kết hợp (*) (4) ta có |A + Bx | ≤ |A + B| − |B| + |Bx | ≤ |A| + |Bx | − 2, tức (4) lại cho Bx , mà Bx ⊂ B, mâu thuẫn với tính nhỏ |B| Vậy điều phản chứng sai Định chứng minh hoàn toàn 2.2 Chứng minh đa thức nhiều biến Bổ đề 2.2 Cho đa thức P(x1 , x2 , , xn ) đa thức n biến hệ số thực với bậc P theo biến xi lớn ti , ≤ i ≤ n Gọi S1 , S2 , , Sn tập R |Si | = ti + 1, ∀i = 1, 2, , n Nếu P(s1 , s2 , , sn ) = với (s1 , s2 , , sn ) ∈ S1 × S2 × · · · × Sn P ≡ Chứng minh Ta chứng minh quy nạp theo n, số biến đa thức P Với n = 1, P(x1 ) đa thức biến x1 , có bậc t1 Vì P(x1 ) = |S1 | = t1 + điểm, nên theo tính chất đa thức biến P ≡ Vậy bổ đề cho n = Giả sử bổ đề cho đa thức n − biến, n ≥ Xét đa thức P(x1, x2 , , xn ) đa thức n biến Ta viết đa thức đa thức biến xn : tn Pn(xn ) = P(x1 , x2 , , xn ) = ∑ Qi (x1 , , xn−1)xin i=0 Vì đa thức Pn đa thức biến xn , có bậc tn tn + điểm Sn Do lại theo tính chất đa thức biến ta Pn ≡ ⇒ Qi (x1 , x2 , , xn−1 ) = 0, ∀i = 0, 1, ,tn (đến chứng tỏ đa thức n − biến Qi (i = 1, 2, ,tn ) điểm S1 × S2 × · · · × Sn−1 Tiếp theo phải chứng minh đa thức đồng 0) Nhưng với i = 0, 1, ,tn Qi (x1 , x2, , xn−1 ) = 0, ∀(x1, x2 , , xn−1 ) ∈ S1 × S2 × · · · × Sn−1 nên theo giả thiết quy nạp Qi ≡ 0, ∀i = 0, 1, ,tn Từ chứng tỏ P ≡ Kết luận toán với n Theo phương pháp quy nạp, bổ đề cho đa thức n biến Định 2.3 Cho đa thức P(x1 , x2, , xn ) đa thức n biến hệ số thực Gọi S1 , S2 , , Sn tập R Định nghĩa đa thức Qi (xi ) = ∏ (xi − s) s∈Si Nếu P(s1 , s2 , , sn ) = với (s1 , s2 , , sn ) ∈ S1 × S2 × · · · × Sn , tồn đa thức R1 , , Rn ∈ R[x1 , , xn ] với deg Ri ≤ deg P − deg Qi cho P = R1 Q + R2 Q + · · · + Rn Q n mn Chứng minh Đặt ti = |Si | − Ta xét thuật toán sau đây: "Nếu có đơn thức cxm xn xn P mi −(i+1).Qi (xi ) mi i mà mi > ti , với số i đó, ta thay đại lượng xm " i xi − xi Ths Trần Minh Hiền(0989.541.123) - Trường THPT chuyên Quang Trung ĐỊNH CAUCHY - DAVENPORT VÀ ỨNG DỤNG ĐỊNH CAUCHY - DAVENPORT mn • Do deg Qi = ti + Qi đa thức monic, đơn thức xm xn xn thay số đơn thức khác, tất đơn thức có bậc nhỏ m1 + m2 + · · · + mn • Do đó, sau bước thuật tốn này, tổng bậc tất đơn thức P giảm thực Nhưng tổng ban đầu hữu hạn, nên thuật toán kết thúc sau số hữu hạn lần thực Gọi P đa thức cuối nhận sau lần thực thuật tốn Khi phép biến đổi mi −(i+1) mi mi m1 mn cxm xi xn xn → cx1 xi − xi n Qi (xi ) xm n tương ứng phép trừ đa thức P cho đa thức có dạng R′i Qi , với mi −(i+1) R′i = cxm xi n xm n deg R′i = m1 + · · · + mn − (ti + 1) ≤ deg P − deg Qi Do q trình thực cho tất đơn thức P, P = P − R1 Q1 + · · · + Rn.Qn (ở Ri tổng đa thức R′i (mỗi đơn thức P có mi > ti ta có R′i Thực thuật toán cho tất đơn thức này, ta R′i khác nhau) Vì R′i có bậc ≤ deg P − deg Qi nên) ta có deg Ri ≤ deg P − deg Qi Vì P nhận cuối thật tốn trên, nên biến xi P có lũy thừa cao ti Vì Qi (xi ) = 0, ∀xi ∈ Si nên suy P(s1 , s1 , , sn ) = P(s1 , s2 , , sn ), ∀(s1 , s2 , , sn ) ∈ S1 × S2 × · · · × Sn Theo bổ đề 2.2 P ≡ 0, P = R1 Q + R2 Q + · · · + Rn Q n Định 2.4 (Combinatorial Nullstellensatz) Cho đa thức P(x1 , , xn ) ∈ R[x1 , , xn ] có bậc deg P = t1 + + tn , với ti số nguyên không âm hệ số đơn thức xt11 xtnn khác Nếu S1 , , Sn tập R thỏa |Si | > ti , tồn (s1, , sn ) ∈ S1 × S2 × · · · × Sn cho P(s1, s2 , , sn ) = Ths Trần Minh Hiền(0989.541.123) - Trường THPT chuyên Quang Trung ĐỊNH CAUCHY - DAVENPORT VÀ ỨNG DỤNG ĐỊNH CAUCHY - DAVENPORT Chứng minh Ta cần chứng minh kết luận toán trường hợp |Si | = ti + (vì |Si | > ti + hiển nhiên kết luận toán đúng, ta chứng minh khẳng định với |Si | = ti + 1) Giả sử kết luận tốn khơng đúng, tức P(s1 , s2 , , sn ) = 0, ∀(s1 , , sn ) ∈ S1 × S2 × · · · × Sn Khi đó, theo định 2.3 P = R1 Q1 + R2 Q2 + · · · + Rn.Qn với deg Ri ≤ deg P − deg Qi , ∀i = 1, 2, , n Theo giả thiết, hệ số xt11 xtnn vế phải khác Tuy nhiên, deg Ri Qi ≤ deg P đơn thức Ri Qi mà có bậc t1 + · · · + tn (theo cách xây dựng 2.3), lấy Ri nhân với xtii +1 khai triển Qi (xi ) = ∏ (x − si) = xtii+1 + s∈Si ··· thừa số lại có bậc ≤ ti Từ suy đơn thức R1 Q1 + · · · + Rn Qn mà có tổng bậc t1 + · · · +tn phải chia hết cho xtii +1 , với số i ∈ {1, 2, , n} Do đơn thức xt11 xtnn nằm R1 Q1 + · · · + Rn Qn có tổng bậc t1 + · · · + tn phải chia hết cho xtii +1 Điều xảy hệ số xt11 xtnn Điều trái với giả thiết Vậy điều phản chứng sai Định chứng minh Chú ý: Các định cho đa thức trường R, trường Z p Trước áp dụng định 2.4 để chứng minh định 2.1 ta thấy ứng dụng đẹp tốn tiếp sau: IMO 2007 Ví dụ 2.1 (IMO 2007) Cho n số nguyên dương Đặt S = {(x, y, z)|x, y, z ∈ {0, 1, , n}, x + y + z > 0} tập chứa (n + 1)3 − điểm không gian ba chiều Xác định số mặt phẳng nhất, cho hợp chúng chứa S, không chứa điểm (0, 0, 0) Chứng minh Dễ dàng nhận thấy 3n mặt phẳng có phương trình x + y + z = i, ∀i = 1, 2, , 3n đạt yêu cầu toán Ta chứng minh 3n số mặt phẳng nhỏ cần tìm Giả sử ngược lại, tồn mặt phẳng P1 , P2, , Pk (k ≤ 3n − 1) mà hợp chúng phủ hết tập S không chứa điểm (0, 0, 0) Mỗi mặt phẳng Pi xác định phương trình x + bi y + ci z + di = Nhận thấy di = (0, 0, 0) ∈ Pi Hợp mặt phẳng Pi phủ hết điểm S k P(x, y, z) = ∏(ai x + bi y + ci z + di ) = 0, ∀(x, y, z) ∈ S i=1 Vì ∈ Pi nên P(0, 0, 0) = Đặt S1 = S2 = S3 = {0, 1, 2, , n} Khi S ∪ {(0, 0, 0)} = S1 × S2 × S3 Ths Trần Minh Hiền(0989.541.123) - Trường THPT chuyên Quang Trung ĐỊNH CAUCHY - DAVENPORT VÀ ỨNG DỤNG Xét đa thức ĐỊNH CAUCHY - DAVENPORT k Q(x, y, z) = P(x, y, z) − c ∏(x − i)(y − i)(z − i), i=1 với c số c = P(0, 0, 0) (−1)3n (n!)3 • Nhận thấy Q(x, y, z) = 0, ∀(x, y, z) S1 ì S2 ì S3 () Mặt khác, k < 3n nên deg Q = 3n Hệ số xn yn zn Q −c = Theo định 2.4, tồn điểm (x0 , y0 , z0 ) ∈ S1 × S2 × S3 để Q(x0 , y0 , z0 ) = 0, mâu thuẫn với (*) Từ chứng tỏ điều phản chứng sai Bài toán chứng minh Chứng minh định 2.1 định 2.4 • Nếu |A| + |B| − ≥ p Khi A + B = Z p Thật vậy, |A| + |B| > p nên với x ∈ Z p tập A ∩ (x − B) = 0/ (vì A ∩ (x − B) = 0/ A, e − B tập Z p nên p ≥ |A| + |x − B| = |A| + |B| > p, vơ lý) Từ phần tử x ∈ Z p viết dạng x = a + b, a ∈ A, b ∈ B hay x ∈ A + B Chứng tỏ Z p ⊂ A + B, suy A + B = Z p , định trường hợp • Nếu |A| + |B| < p Giả sử kết luận tốn khơng đúng, tức |A + B| ≤ |A| + |B| − Đặt C tập hợp cho A + B ⊂ C |C| = |A| + |B| − Xét đa thức P(x, y) = ∏ (x + y − c) c∈C deg P = |A| + |B| − Vì A + B ⊂ C nên P(a, b) = 0, ∀(a, b) ∈ A × B (∗) Vì |A| + |B| − < p nên hệ số khác Mặt khác, hệ số x|A|−1 y|B|−1 P |A|+|B|−2 |A|−1 trong Z p Do theo định 2.4 tồn (a, b) ∈ A × B cho P(a, b) = 0, mâu thuẫn với (*) Vậy điều phản chứng không Chứng tỏ kết luận trường hợp Định 2.5 Cho p số nguyên tố, A, B hai tập khác rỗng Z p Định nghĩa A ⊕ B = {a + b|a ∈ A, b ∈ B, a = b} Khi |A ⊕ B| ≥ min{p, |A| + |B| − 3} (1) Nếu |A| = |B| ta có kết mạnh |A ⊕ B| ≥ min{p, |A| + |B| − 2} (2) Chứng minh • Nếu |A| = |B| = 1, bất đẳng thức (2) hiển nhiên Xét |A|, |B| ≥ Trong trường hợp |A = |B|, ta lấy tập B′ = B\{b}, với b phần tử B Khi áp dụng (2) cho hai tập A B′ ta bất đẳng thức (1) Ths Trần Minh Hiền(0989.541.123) - Trường THPT chuyên Quang Trung ĐỊNH CAUCHY - DAVENPORT VÀ ỨNG DỤNG ĐỊNH CAUCHY - DAVENPORT • Từ ta cần chứng minh (2) – Trường hợp |A| + |B| − ≥ p Nếu p ≥ hiển nhiên Xét p > p lẻ Ta chứng minh A + B = Z p Thật vậy, xét g ∈ Z p phần tử tùy ý Đặt C = g − B, |C| = |B| Khi |A| + |C| = |A| + |B| ≥ p + Theo công thức |X ∪Y | + |X ∩Y | = |X| + |Y | ta có |Z p | + |A ∩C| ≥ |A ∪C| + |A ∩C| = |A| + |C| ≥ p + ⇒ |A ∩C| ≥ Gọi x, y hai phần tử phân biệt A ∩C Vì C = g − B, nên tồn hai phần tử bx , by ∈ B cho x + bx = y + by = g Ta có kết luận toán x = bx y = by (khi g ∈ A ⊕ B) Giả sử hai điều không xảy ra, x + x = y + y = g ⇒ 2(x − y) = Suy p, vô p nguyên tố lẻ – Xét trường hợp |A| + |B| − < p Giả sử kết luận định không đúng, tức |A ⊕ B| < |A| + |B| − Gọi C tập hợp Z p có |A| + |B| − phần tử A + ⊕B ⊂ C Xét đa thức P(x, y) = (x − y) ∏ (x + y − c) c∈C Nhận thấy với a ∈ A, b ∈ B P(a, b) = Mặt khác, ta có deg P = |C| + = |A| + |B| − Do |A| = |B|, nên hệ số x|A|−1 y|B|−1 P (m + n − 3)! m+n−3 m+n−3 (m + n − 3)! = − − m−1 m−2 (m − 2)!(n − 1)! (m − 1)!(n − 2)! (m + n − 3)! = (m − n), (m − 1)!(n − 2)! khác (theo modulo p) m + n − < p m = n Do theo định 2.4 tồn (a, b) ∈ A × B cho P(a, b) = 0, mâu thuẫn Vậy điều giả sử sai Trường hợp chứng minh Vậy định chứng minh hoàn toàn Định 2.6 (Định Cauchy - Davenport tổng quát) Giả sử p số nguyên tố A1 , A2 , , An tập Z p (n ≥ 2) Khi |A1 + A2 + · · · + An | ≥ min{p, |A1| + |A2 | + · · · + |An | − (n − 1)} Chứng minh Ta chứng minh phương pháp quy nạp Với n = 2, kết định 2.1 Giả sử định với n − Khi đó, với n tập A1 , , An |A1 + A2 + · · · + An | = |(A1 + + An−1 ) + An )| ≥ min{p, |A1 + + An−1 | + |An | − 1} (áp dụng cho n = tập) ≥ min{p, min{p, |A1| + |A2 | + + |An−1 | − (n − 2)} + |An| − 1} (giả thiết quy nạp cho n − tập) Ths Trần Minh Hiền(0989.541.123) - Trường THPT chuyên Quang Trung ĐỊNH CAUCHY - DAVENPORT VÀ ỨNG DỤNG MỘT SỐ ỨNG DỤNG - Nếu min{p, |A1 | + |A2 | + + |An−1 | − (n − 2)} = p(∗), min{p, min{p, |A1 |+|A2 |+ .+|An−1 |−(n−2)}+|An|−1} = min{p, p+|An |−1} = p(1)(do |An | ≥ 1) Mặt khác từ (*) |A1 | + |A2 | + + |An−1 | − (n − 2) ≥ p nên |A1| + |A2 | + + |An−1| + |An | − (n − 1) ≥ p + |An | − ≥ p ⇒ min{p, |A1 | + |A2 | + · · · + |An | − (n − 1)} = p (2) Từ (1) (2) suy min{p, min{p, |A1 |+|A2 |+ .+|An−1 |−(n−2)}+|An|−1} = min{p, |A1 |+|A2 |+· · ·+|An |−(n−1)}(3) - Nếu min{p, |A1 | + |A2 | + + |An−1 | − (n − 2)} = |A1 | + |A2| + + |An−1 | − (n − 2) min{p, min{p, |A1 | + |A2 | + + |An−1 | − (n − 2)} + |An| − 1} = min{p, |A1 | + |A2 | + + |An−1 | + |An | − (n − 1)} (4) Trong hai trường hợp (3) (4) ta có |A1 + A2 + · · · + An | ≥ min{p, |A1 | + |A2 | + · · · + |An | − (n − 1)} tức định cho n Theo phương pháp quy nạp ta có điều phải chứng minh Định 2.7 (Định Vosper) Cho p số nguyên tố A, B hai tập Z p cho |A|, |B| ≥ |A + B| ≤ p − Khi |A + B| = |A| + |B| − A, B hai cấp số cộng có cơng sai Chú ý: Bạn đọc cần tìm hiểu chứng minh định này, tham khảo chứng minh sơ cấp định 3.2 tài liệu tham khảo [4] Ngoài tài liệu này, định 3.1 cung cấp thêm cách chứng minh sơ cấp quy nạp, theo qua phép biến đổi e Chính cách chứng minh cho ta lời giải cho tập 4.11 MỘT SỐ ỨNG DỤNG Ví dụ 3.1 (POLAND 2003) Chứng minh với số nguyên tố p > 3, tồn số nguyên x, y, k cho < 2k < p x2 + y2 = kp + Giải Kết tốn khơng thay đổi thay x, y p − x, p − y Do ta số x, y p miền ≤ x, y < thỏa mãn điều kiện toán Khi x, y nằm miền hiển nhiên phần thương k thỏa 2k < p, p2 p2 p2 2 x +y < + = 4 p Đặt S tập chứa tất bình phương theo modulo p Chú ý x2 ≡ (p−x)2 ( mod p), ∀x ∈ 1, 2, , p+1 Do |S| = Theo định 2.1 |S + S| ≥ min{p, 2|S| − 1} = p, p+1 − = p ⇒ |S + S| = p Ths Trần Minh Hiền(0989.541.123) - Trường THPT chuyên Quang Trung 10 ĐỊNH CAUCHY - DAVENPORT VÀ ỨNG DỤNG MỘT SỐ ỨNG DỤNG Điều chứng tỏ thặng dư modulo p viết thành tổng hai số phương Và tốn trường hợp đặc biệt với kết Ví dụ 3.2 Cho p số nguyên tố Cho trước p − số ngun cho khơng có số chia hết cho p Chứng minh ta đổi dấu vài số chúng để tổng số thu chia hết cho p Chứng minh Gọi p − số nguyên a1 , a2 , , a p−1 Với i = 1, 2, , p − 1, đặt Ai = {ai , −ai }( mod p) Khi |Ai | = 2, ∀i = 1, 2, , p − ≡ −ai ( mod p) Áp dụng định 2.6 ta có |A1 + A2 + · · · + A p−1 | ≥ min{p, |A1 | + · · · + A p−1 − (p − 2)} = min{p, p} = p Từ suy |A1 + A2 + · · · + A p−1 | = p hay tập A1 + A2 + · · · + A p−1 chứa hệ thặng dư đầy đủ modulo p Đây kết luận tốn Ví dụ 3.3 Giả sử p > số nguyên tố cho p − số nguyên a1 , a2 , , a p−1 thỏa mãn a1 a p−1 p a1 + a2 + · · · + a p−1 p Chứng minh chia tập thành hai nhóm rời để tổng phần tử hai nhóm đồng dư với theo modulo p Chứng minh Đặt Ai = {0, }( mod p) Thì p, ∀i = 1, 2, , p − nên |Ai | = 2, ∀i = 1, 2, , p − Theo định 2.6 |A1 + A2 + · · · + A p−1| ≥ min{p, |A1 | + · · · + |A p−1 | − (p − 2)} = min{p, 2(p − 1) − (p − 2)} = p Do A1 + A2 + · · · + A p−1 = Z p Khi tồn bi ∈ Ai , i = 1, 2, , p − cho b1 + b2 + · · · + b p−1 ≡ a1 + a2 + · · · + a p−1 ( mod p) Vì phần tử bi ∈ Ai bi 0, Tuy nhiên a1 + a2 + · · · + a p−1 ≡ 0( mod p) nên tồn số phần tử bi khác Gọi phần tử a j1 , , a jk , thay vào ta có a j1 + · · · + a jk ≡ a1 + a2 + · · · + a p−1 ( mod p) hay a j1 + · · · + a jk ≡ a1 + · · · + a p−1( mod p) hay a j1 + · · · + a jk ≡ (a1 + · · · + a p−1 ) − (a j1 + · · · + a jk )( mod p) toán chứng minh Ví dụ 3.4 (USAMO 2009) Cho n số nguyên dương Xác định tập A lớn tập {−n, −n + 1, , n − 1, n} cho A không tồn ba phần tử a, b, c (không thiết phân biệt) mà a + b + c = Ths Trần Minh Hiền(0989.541.123) - Trường THPT chuyên Quang Trung 11 ĐỊNH CAUCHY - DAVENPORT VÀ ỨNG DỤNG MỘT SỐ ỨNG DỤNG Phân tích hướng dẫn giải Nhận xét ∈ A (vì ∈ A = + + 0, mâu thuẫn với tính chất tập A) Đặt A+ = A ∩ {1, 2, , n}, A− = A ∩ {−n, −n + 1, , −1} • Nhận xét A+ + A− −A hai tập rời {−n, −n + 1, , n − 1, n} Thật vậy, / tồn số c mà (A+ + A− ) ∩ (−A) = 0, c = i + j = −k (i ∈ {1, 2, , n} ∩ A, j ∈ {−n, −n + 1, , −1} ∩ A, k ∈ A) ⇒ i + j + k = 0, mâu thuẫn với định nghĩa tập A • Vì A+ + A− −A hai tập rời tập {−n, −n + 1, , n − 1, n} (tập có 2n + phần tử) nên |A+ + A− | + | − A| ≤ 2n + ⇒ 2n + ≥ |A+ + A− | + |A| =|A| Ngoài theo định 1.2 |A+ + A− | ≥ |A+ | + |A− | − Do 2n + ≥ |A+ | + |A− | −1 + |A| = 2|A| − ⇒ |A| ≤ n + =|A| Đến dự đốn tập A lớn có n + phần tử Việc cần tập A có n + phần tử Để thỏa mãn toán, ta cần lấy tập A phần tử lớn nhỏ tập đề Đó tập         n n A = −n, −n + 1, , − − 1, + 1, , n   2     phần âm phần dương Lấy ba phần tử a, b, c tùy ý tập A Nếu ba phần tử thuộc phần âm, phần dương tập A hiển nhiên a + b + c = Nếu hai số thuộc phần dương, giả sử a, b, số c thuộc phần âm, c ≥ −n a+b ≥ n n +1+ +1 = 2 n +1 > 2· n = n ≥ c ⇒ a + b + c > Tương tự kiểm tra hai số thuộc phần âm, số thuộc phần dương tập A thỏa mãn đề Trở ngại lại tập A có n + phần tử n lẻ Còn n chẵn, tập A có n phần tử, không đạt tới n + phần tử Tuy nhiên ta n chẵn tập A khơng thể có n + phần tử Thật vậy, giả sử |A| = n + với n chẵn Do A− ⊂ {−n, , −1} A+ ⊂ {1, 2, , n} nên A+ + A− ⊆ {−n + 1, −n + 2, , n − 2, n − 1} Ths Trần Minh Hiền(0989.541.123) - Trường THPT chuyên Quang Trung 12 ĐỊNH CAUCHY - DAVENPORT VÀ ỨNG DỤNG MỘT SỐ ỨNG DỤNG Lại A+ + A− −A hai tập hợp rời tập 2n + phần tử {−n, −n + 1, , n − 1, n} nên 2n + = |A+ + A− | + | − A| xảy −n, n ∈ −A, tức −n, n ∈ A Vì −n ∈ A, theo cấu trúc tập A định nghĩa tập A+ , cặp phần tử sau (n chẵn) n n n − 1, + , {1, n − 1}, {2, n − 2}, , 2 n n có nhiều phần tử cặp thuộc vào A+ Dẫn đến |A|+ ≤ , tương tự |A− | ≤ 2 Do n n |A| = |A+ | + |A− | ≤ + = n 2 mâu thuẫn với |A| = n + Vậy toán chứng minh hồn tồn Ví dụ 3.5 (VIETNAM TST 2012) Cho số nguyên tố p ≥ 17 Chứng minh t = số nguyên dương lớn thỏa mãn điều kiện: Với số nguyên a, b, c, d cho số abc không chia hết p cho p a + b + c chia hết cho p tồn số nguyên x, y, z thuộc tập 0, 1, 2, − cho t ax + by + cz + d p Chứng minh Nhận xét Nếu {u1 , , uk } chứa hệ thặng dư đầy đủ modulo p tập d +{u1, , uk } chứa hệ thặng dư đầy đủ modulo p p − 1, A = {0, 1, 2, , k} Đặt Đặt k = t S = {ax + by + cz|0 ≤ x, y, z ≤ k} = a.A + b.A + c.A( mod p) • Với t = 3, theo yêu cầu toán nhận xét tốn tương đương với việc chứng minh t = S chứa hệ thặng dư đầy đủ modulo p, tức cần chứng minh |S| = p Đặt C = {ax + by + cz| − ≤ x, y, z ≤ 1} Do a + b + c ≡ 0( mod p) nên với (x, y, z ∈ A) ax + by + cz ≡ a(x − k) + b(y − k) + c(z − k)( mod p) Tức (x, y, z) ∈ A tương ứng với (x − k, y − k, z − k), nên ta thực đối xứng tập k A để sử dụng tập C, cách đặt L = xét tập S′ = {ax + by + cz| − L ≤ x, y, z ≤ L} |S| ≥ |S′ | (dấu xảy k chẵn) • Khi tập S′ viết dạng S′ = C +C + · · · +C L lần Theo định 2.6 ta có |S′ | ≥ min{p, L.|C| − (L − 1)} (∗) Để có kết luận tốn |S| = p, ta cần |S′ | = p, theo (*) ta cần chứng minh L.|C| − (L − 1) ≥ p (∗∗) Ths Trần Minh Hiền(0989.541.123) - Trường THPT chuyên Quang Trung 13 ĐỊNH CAUCHY - DAVENPORT VÀ ỨNG DỤNG MỘT SỐ ỨNG DỤNG • Với ý a, b, c không chia hết cho p nên tập {0, a, b, c, −a, −b, −c, a − b, b − c, c − a} gồm 10 phần tử đôi phân biệt theo modulo p nằm tập C Do |C| ≥ 10 Vậy để có (**) ta chứng minh 10.L − (L − 1) ≥ ⇔ 9L + ≥ p (∗ ∗ ∗) Vì p nguyên tố, nên xét hai trường hợp – p dạng p = 6a + Do p ≥ 17 nên a ≥ Khi k = a− = a − Do (***) tương đương 9(a − 1) + ≥ 6a + ⇒ a ≥ p − = 2a − 1, nên L = ⇒a≥2 = (đúng) p − = 2a, nên L = – p dạng p = 6a + Do p ≥ 17 nên a ≥ Khi k = (***) tương đương 9(a) + ≥ 6a + ⇔ a ≥ k k = a Do (đúng, a nguyên) Vậy trường hợp p (***) Vậy tốn chứng minh cho t = • Ta chứng minh t = không thỏa mãn Thật vậy, chọn a = b = 1, c = −2 d = với x, y, z ∈ A = 0, 1, 2, , −2 p − p p−1 p−1 ≤ x + y − 2z + −1 + ≤2 2 x=y=0,z=[ 4p ]−1 p−1 Khi p−1 p −1 + x=y=[ 4p ]−1,z=0 hay p−1 ≤ x + y − 2z + ≤ p− 2 Trong khoảng giá trị mãn , p− 2 khơng có số chia hết cho p, tức khơng tồn x, y, z thỏa Ví dụ 3.6 (IMO SHORTLIST 2007) Cho X tập hợp gồm 10000 số ngun, khơng có số chúng chia hết cho 47 Chứng minh tồn tập Y X gồm 2007 phần tử cho a − b + c − d + e 47, ∀a, b, c, d, e ∈ Y Phân tích hướng dẫn giải d + e, ∀a, b, c, e, d ∈ M • Một tập M gồm số nguyên gọi tốt 47 ∤ a − b + c − • Nhận thấy tập J = {−9, −7, −5, −3, −1, 1, 3, 5, 7, 9} tập tốt Thật vậy, với phần tử a, b, c, d, e ∈ J số a − b + c − d + e số lẻ −45 = (−9) − + (−9) − + (−9) ≤ a − b + c − d + e ≤ − (−9) + − (−9) + = 45 số nguyên lẻ chia hết cho 47 nằm tập {−45, −43, , 43, 45} Ths Trần Minh Hiền(0989.541.123) - Trường THPT chuyên Quang Trung 14 ĐỊNH CAUCHY - DAVENPORT VÀ ỨNG DỤNG MỘT SỐ ỨNG DỤNG • Với k = 1, 2, , 46, tập hợp Ak = {x ∈ X|∃ j ∈ J : kx ≡ j( mod 47)} tập tốt Thật vậy, giả sử tồn tập Ak không tốt (k ∈ {1, 2, , 46}) Khi tồn phần tử x1 , x2, x3 , x4 , x5 ∈ Ak cho x1 − x2 + x3 − x4 + x5 ≡ 0( mod 47) ⇒ k(x1 − x2 + x3 − x4 + x5) ≡ 0( mod 47) Dẫn đến kx1 − kx2 + kx3 − kx4 + kx5 ≡ 0( mod 47) Theo định nghĩa x1 , , x5 , tồn j1 , , j5 ∈ J cho j1 − j2 + j3 − j4 + j5 ≡ kx1 − kx2 + kx3 − kx4 + kx5 ≡ 0( mod 47) chứng tỏ J tập khơng tốt, mâu thuẫn • Mỗi phần tử x ∈ X thuộc 10 tập Ak Thật vậy, 47 số nguyên tố, tập {1, 2, , 46} lập thành hệ thặng dư thu gọn modulo 47, x ∈ X nên (x, 47) = 1, {x, 2x, , 46x} lập thành hệ thặng dư thu gọn modulo 47 Do với j ∈ J (gồm có 10 giá trị j) tương ứng tồn 10 số k ∈ {1, 2, , 46} (các j khác k khác nhau) để kx ≡ j( mod 47) Do x thuộc vào 10 tập Ak (k ∈ {1, 2, , 47}) • Do ∑ 6|Ak| = 10|X| = 100000 k=1 Theo nguyên trung bình, tồn số k cho |Ak | ≥ 100000 > 2173 > 2007, 46 tức tập Ak tập cần tìm Trong lời giải trên, ta khơng thấy sử dụng định Cauchy - Davenport Hãy nhìn kỹ lời giải trên, bước tập J quan trọng Tập J tập tốt có 10 phần tử Thực tập chứa số dư khác chia cho 47 mà tập tốt chứa tối đa 10 phần tử Thật vậy, |J| ≥ 11, theo định 2.6 thì, 5|J| − ≥ × 11 − = 51 > 47 |J + J + J + (−J) + (−J)| ≥ min{5|J| − 4, 47} = 47 tồn a, b, c, d, e ∈ J cho a − b + c − d + e 47, suy tập J không tốt Vậy |J| ≤ 10 Để |J| = 10 theo định 2.7, J phải cấp số cộng Ví dụ 3.7 (Định Erdos - Ginzburg - Ziv) Cho n ≥ a0 , a2 , , a2n−2 dãy gồm 2n − số nguyên (không thiết phân biệt) Chứng minh tồn dãy gồm n số ai1 , , ain cho ai1 + ai2 + · · · + ain ≡ 0( mod n) Phân tích hướng dẫn giải • Trước tiên ta chứng minh kết luận toán trường hợp n = p ′ số nguyên tố Gọi ∈ Z, ≤ a′i < p cho ≡ a′i ( mod p) (thực lấy modulo phần tử dãy) Ta đánh lại số cho số để ≤ a′0 ≤ a′1 ≤ ≤ a′2p−2 ≤ p − Ths Trần Minh Hiền(0989.541.123) - Trường THPT chuyên Quang Trung 15 ĐỊNH CAUCHY - DAVENPORT VÀ ỨNG DỤNG MỘT SỐ ỨNG DỤNG – Nếu a′i ≡ a′i+p−1 với số i ∈ [1, p − 1] theo thứ tự xảy a′i ≡ a′i+1 ≡ · · · ≡ a′i+p−1 ( mod p) dẫn đến + ai+1 + · · · + ai+p−1 ≡ pai ≡ 0( mod p) – Nếu a′i ≡ a′i+p−1 với i ∈ [1, p − 1] Khi đặt Ai = {a′i , a′i+p−1 } |Ai | = 2, ∀i = 1, 2, , p − Áp dụng định 2.6 ta có |A1 +A2 +· · ·+A p−1 | ≥ min{p, |A1 |+· · ·+|A p−1 |−(p−2)} = min{p, 2(p−1)−(p−2)} = p Từ suy A1 + A2 + · · · + A p−1 = Z p Từ tồn lớp đồng dư a′ji ∈ Ai , i = 1, 2, , p − 1, với ji ∈ {i, i + p − 1} cho −a0 ≡ a′j1 + a′j2 + · · · + a′j p−1 ( mod p) Suy a j1 + a j2 + · · · + a j p−1 + a0 ≡ 0( mod p) Vậy toán trường hợp n ngun tố • Ta chứng minh kết luận tốn trường hợp tổng quát phương pháp quy nạp Nếu n = 1, toán hiển nhiên Giả sử với n > kết luận toán cho số nguyên dương nhỏ n Ta chứng minh toán với n Nếu n nguyên tố, toán theo ý Nếu n hợp số, đặt n = u.v (1 < u ≤ v < n) Khi theo giả thiết quy nạp, toán cho u v – Từ dãy số a0 , , a2n−2 có độ dài 2n − = 2uv − 1, theo giả thiết quy nạp, tồn dãy a1,i1 , , a1,iv cho a1,i1 + · · · + a1,iv ≡ 0( mod v) – Khi lại 2n − − v = (2u − 1)v − số nguyên nằm dãy ban đầu, không thuộc vào dãy Do 2u − ≥ 2, lại áp dụng giả thiết quy nạp, từ dãy độ dài (2u − 1)v − ≥ 2v − số nguyên, tồn dãy a2,i1 , , a2,iv cho a2,i1 + · · · + a2,iv ≡ 0( mod v) – Khi lại 2n − − 2v = (2u − 2)v − số nguyên dãy không thuộc vào hai dãy Cứ tiếp tục trình trên, với j = 1, 2, , 2u − 1, ta nhận 2u − dãy đôi rời a j,i1 , , a j,iv độ dài v cho a j,i1 + · · · + a j2,iv ≡ 0( mod v) Khi a j,i1 + · · · + a j2,iv = b j v(b j ∈ Z), ∀ j = 1, 2, , 2u − Ths Trần Minh Hiền(0989.541.123) - Trường THPT chuyên Quang Trung 16 ĐỊNH CAUCHY - DAVENPORT VÀ ỨNG DỤNG MỘT SỐ ỨNG DỤNG – Do giả thiết cho u, nên từ dãy b1 , b2 , , b2u−1 độ dài 2u − có dãy b j1 , , b ju cho b j1 + b j2 + · · · + b ju ≡ 0( mod u), tức b j1 + b j2 + · · · + b ju = c.u(c ∈ Z) Khi u ∑ v ∑ a jr ,is = r=1 s=1 u ∑ b jr v = cuv = cn ≡ 0( mod n) r=1 Định chứng minh hồn tồn Ví dụ 3.8 (CHINA TST 2016) Cho m, n số nguyên dương thỏa mãn n ≥ m ≥ S tập hợp gồm có n số tự nhiên Chứng minh S chứa 2n−m+1 tập phân biệt, mà tập có tổng phần tử chia hết cho m (tập rỗng xem tập thỏa mãn) Trước chứng minh, ta cần sử dụng số kết quen thuộc cho nhận xét sau: Nhận xét 1: Trong n số nguyên tùy ý, tồn số vài số mà tổng chúng chia hết cho n Đây tính chất quen thuộc chứng minh Dirichlet Nhận xét 2: Cho p số nguyên tố Q = {a1 , a2 , , a p−1} tập hợp chứa số nguyên không chia hết cho p Khi tập R = {S(A) = ∑ x|A ⊂ Q} x∈A chứa hệ thặng dư đầy đủ modulo p Ở S(0) / =0 Chứng minh Ta lấy phần tử Q theo modulo p mà khơng làm thay đổi tốn Xét tập A1 = {0, a1 }, A2 = {0, a2 }, , A p−1 = {0, a p−1 } p, ∀i = 1, 2, , p − nên |Ai | = 2, i = 1, 2, , p − Theo định 2.6 |A1 + A2 + · · · + A p−1| ≥ min{p, |A1 | + · · · + |A p−1 | − (p − 2)} = min{p, 2(p − 1) − (p − 2)} = p Chứng tỏ A1 + · · · + A p−1 = Z p Mặt khác, phần tử A1 + A2 + · · · + A p−1 phần tử R (phần tử + + · · · + A1 + · · · + A p−1 ứng với việc chọn tập A = 0/ ⊂ Q) Chứng minh Ý tưởng toán ta chia tập S thành hai tập A, B Mỗi tập A′ A tìm tập B′ B mà tổng số hai tập chia hết cho m Đồng nghĩa với việc S(A′ ) ≡ −S(B′)( mod m) • Trước tiên ta chứng minh m = p nguyên tố, toán hiển nhiên Thật vậy: – Nếu S có p − số ngun không chia hết cho m Gọi Q = {a1 , a2 , , a p−1 } tập chứa p − số nguyên không chia hết cho p Khi S = Q ∪ (S\Q) Tập S\Q có n − (p − 1) = n − p + phần tử nên có 2n−p+1 tập Mỗi tập A ⊂ (S\Q), nhận xét 2, tồn tập B ⊂ Q cho S(A) ≡ −S(B)( mod p) ⇒ S(A) + S(B) ≡ 0( mod p) ⇒ S(A ∪ B) ≡ 0( mod p) Ths Trần Minh Hiền(0989.541.123) - Trường THPT chuyên Quang Trung 17 ĐỊNH CAUCHY - DAVENPORT VÀ ỨNG DỤNG MỘT SỐ ỨNG DỤNG Vậy tập A ⊂ (S\Q), ln tìm tập dạng A ∪ B ⊂ S để S(A ∪ B) p Mà có 2n−p+1 tập A nên tương ứng có 2n−p+1 tập S thỏa mãn đề – Nếu S có < p − số ngun khơng chia hết cho m Dẫn đến S có ≥ n − p + số chia hết cho m Trong trường hợp hiển nhiên tập tập chứa n − p + số chia hết cho p, tức có 2n−p+1 tập thỏa mãn tốn • Trong trường hợp m tùy ý Ta chứng minh khẳng định toán quy nạp Với m = toán hiển nhiên Giả sử toán cho giá trị nguyên dương < m Xét với số nguyên dương m Đặt m = p.k, với p nguyên tố k ≥ Một tập A ⊂ S gọi tập "gần tốt tối thiểu" S(A) k S(A) m |A| nhỏ Khi theo nhận xét tập A gần tốt tối thiểu |A| ≤ k – Nếu tập S có tối đa t < p − tập gần tốt tối thiểu Gọi tập S1 , , St Khi xét phân hoạch S S = S1 ∪ S2 ∪ ∪ St ∪ N (N = S\(S1 ∪ S2 ∪ ∪ St )) Vì tập |Si | có tối đa k phần tử Nên tập N có |N| ≥ n − t.p > n − (p − 1)k = n − m + k ≥ k Áp dụng giả thiết quy nạp, tập N chứa 2|N|−k+1 ≥ 2n−m+1 tập phân biệt chia hết cho k Tuy nhiên có tốt đa t tập gần tốt tối thiểu, 2n−m+1 tập chia hết cho k, không tập gần tốt tối thiểu, tức tập có tổng chia chết cho m Kết luận toán chứng minh trường hợp – Nếu tập S có ≥ p − tập gần tốt tối thiểu Chọn p − tập gần tốt tối thiểu phân biệt, ký hiệu B1 , , B p−1 Khi xét phân hoạch S S = B1 ∪ B2 ∪ ∪ B p−1 ∪ M (M = S\(S1 ∪ S2 ∪ ∪ St )) Vì tập |Si | có tối đa k phần tử Nên tập M có |M| ≥ n − (p − 1)k = n − m + k ≥ k Áp dụng giả thiết quy nạp, tập M chứa 2|M|−k+1 ≥ 2n−m+1 tập phân biệt chia hết cho k Do định nghĩa B1 , , B p−1 , đặt S(B1 ) = r1 k, S(B2 ) = r2 k, ., S(B p−1) = r p−1 k (ri ∈ Z+ , i = 1, 2, , p − 1) ri p, ∀i = 1, 2, , p − Đặt Q = {r1 , ,t p−1 }, theo nhận xét 2, tập R = {S(A)|A ⊂ Q} Ths Trần Minh Hiền(0989.541.123) - Trường THPT chuyên Quang Trung 18 ĐỊNH CAUCHY - DAVENPORT VÀ ỨNG DỤNG BÀI TẬP TỰ LUYỆN chứa hệ thặng dư đầy đủ modulo p Do tập S(B1 + B2 + · · · + B p−1) chứa hệ thặng dư {k, 2k, , pk}( mod m) Do lấy tập A ⊂ M (khi S(A) k), ln tồn tập B ⊂ B1 + · · · + B p−1 cho S(B) ≡ −S(A)( mod m) ⇒ S(A ∪ B) ≡ 0( mod m) tức tập A ∪ B thỏa mãn Vì có ≥ 2n−m+1 tập A ⊂ M mà S(A) k, nên có ≥ 2n−m+1 tập thỏa mãn yêu cầu toán Tổng hợp kết trên, tốn chứng minh hồn tồn Lời giải cho trường hợp m = p trường hợp riêng cho lời giải m tổng quát Tuy nhiên trình bày đây, thấy cách làm với m tổng quát, dựa vào cách làm với m = p nguyên tố BÀI TẬP TỰ LUYỆN BÀI TẬP 4.1 (BOSNIA AND HERZEGOVINA TST 2012) Chứng minh với số nguyên tố p lẻ, tồn số nguyên dương m < p số nguyên x1 , x2 , x3 cho x21 + x22 + x23 = mp BÀI TẬP 4.2 Cho k, n số nguyên dương p số nguyên tố Chứng minh tồn số nguyên a1 , , ak cho ak1 + ak2 + · · · + akk ≡ n( mod p) BÀI TẬP 4.3 (BELARUS MO) Cho số nguyên tố p gọi x, y, z ∈ {0, 1, 2, , p − 1} thỏa mãn x2 + y2 + z2 p Gọi S(p) số ba (x, y, z) thỏa mãn điều kiện Chứng minh S(p) ≥ 2p − BÀI TẬP 4.4 (UKRAINIAN TST 2007) Tìm tất số nguyên tố p, cho tồn số nguyên n, để phương trình sau vơ nghiệm ngun x3 + y3 ≡ n( mod p) BÀI TẬP 4.5 (TUYMAADA 2008) Cho 250 số nguyên dương không vượt 501 Chứng minh với số nguyên dương t, tồn bốn số nguyên a1 , a2 , a3 , a4 250 số cho mà a1 + a2 + a3 + a4 − t 23 BÀI TẬP 4.6 Cho p > số nguyên tố Tập {1, 2, , p − 1} phân hoạch thành tập rời A, B,C Chứng minh tồn x, y, z thuộc ba tập A, B,C cho x + y − z p BÀI TẬP 4.7 (APMO 2014) Tìm tất số nguyên dương n cho với số nguyên k, tồn số nguyên a cho a3 + a − k n BÀI TẬP 4.8 (CRUX 2013, VOL 39, CC9) Cho k ≥ số nguyên Đặt n = Zn Chứng minh S + S = Zn Ths Trần Minh Hiền(0989.541.123) - Trường THPT chuyên Quang Trung k(k + 1) S tập 19 ĐỊNH CAUCHY - DAVENPORT VÀ ỨNG DỤNG BÀI TẬP TỰ LUYỆN BÀI TẬP 4.9 (IMO 2003 (general)) Cho n số nguyên dương ≥ Cho A tập chứa n + phần tử tập S = {1, 2, , n3 } Chứng minh tồn n phần tử t1 ,t2 , ,tn S cho tập A j = {x + t j |x ∈ A}, ∀ j = 1, 2, , n đôi rời BÀI TẬP 4.10 (USA TSTST 2013) Cho p số nguyên tố Cho graph G đầy đủ 1000p đỉnh Trên cạnh graph ta gán số nguyên Chứng minh G tồn chu trình mà tổng số đánh cạnh nằm chu trình chia hết cho p BÀI TẬP 4.11 (ROMANIAN TST 2010) Cho X,Y tập hữu hạn nửa khoảng [0, 1) cho ∈ X ∩ Y không tồn x ∈ X, y ∈ Y cho x + y = Chứng minh tập {x + y − [x + y] : x ∈ X, y ∈ Y } có |X| + |Y | − phần tử TÀI LIỆU THAM KHẢO A.Frimu and M.Teleuca, Applications of combinatorial Nullstellensatz, Gazeta Mathematica 2011 O.J.Rodseth, Sumsets mod p, lecture notes M.B.Nathanson, Additive number theory - Inverse problems and the Geometry of Sumset, Springer, 1996 H.Lee, Combinatorial Number Theory, lecture notes Website: http://artofproblemsolving.com Ths Trần Minh Hiền(0989.541.123) - Trường THPT chuyên Quang Trung 20 ... Trường THPT chuyên Quang Trung ĐỊNH LÝ CAUCHY - DAVENPORT VÀ ỨNG DỤNG ĐỊNH LÝ CAUCHY - DAVENPORT xảy ra.) Vậy ta có ∈ B, |B| ≥ 2, A + B = Z p Do tồn phép biến đổi Davenport Bx B để (*) xảy Do ≤ |Bx... có cơng sai ĐỊNH LÝ CAUCHY - DAVENPORT Trong phần này, khơng thích thêm tất tập hợp lấy tập trường Z p , tập chứa hệ thặng dư đầy đủ modulo p Định lý 2.1 (Định lý Cauchy - Davenport) Cho p số... Ths Trần Minh Hiền(0989.541.123) - Trường THPT chuyên Quang Trung ĐỊNH LÝ CAUCHY - DAVENPORT VÀ ỨNG DỤNG ĐỊNH LÝ CAUCHY - DAVENPORT Và hoàn toàn tương tự ta có A + B = A + 2B = A + 3B = · · · Nhưng

Ngày đăng: 03/05/2018, 12:40

TỪ KHÓA LIÊN QUAN

TÀI LIỆU CÙNG NGƯỜI DÙNG

TÀI LIỆU LIÊN QUAN

w